Đến nội dung

xuanhoan23112002 nội dung

Có 95 mục bởi xuanhoan23112002 (Tìm giới hạn từ 26-04-2020)



Sắp theo                Sắp xếp  

#706634 [TOPIC] SỐ HỌC ÔN TẬP THPT CHUYÊN TOÁN 10 NĂM HỌC 2018-2019

Đã gửi bởi xuanhoan23112002 on 22-04-2018 - 09:04 trong Tài liệu - Đề thi

Bài 73: Ta chứng minh bài toán bằng phương pháp phản chứng

Thật vậy ta có thể giả sử a+b là số nguyên tố 

Theo giả thiết ta có: $(a+b)(b+c)(c+a)-8abc \vdots a+b$

Hay $8abc \vdots a+b$. Lại có a+b là số lẻ nên gcd(a+b,8)=1

Do đó $abc \vdots a+b$ 

Mà a+b là số nguyên tố nên xảy ra 1 trong 3 trường hợp sau: $a \vdots a+b$ hoặc $b \vdots a+b$ hoặc $c \vdots a+b$ (điều này là vô lí do a, b, c là 3 cạnh của tam giác nên max{a, b, c}< a+b)

Nên ta có điều giả sử là sai.

Vậy a+b phải là số nguyên tố




#706251 [TOPIC] SỐ HỌC ÔN TẬP THPT CHUYÊN TOÁN 10 NĂM HỌC 2018-2019

Đã gửi bởi xuanhoan23112002 on 17-04-2018 - 23:15 trong Tài liệu - Đề thi

Lời giải của mình cho bài 35 như sau:

PT đã cho $\Leftrightarrow y^3=(x^3+1)(x^2+1)$

Do x là số lẻ ta dễ dàng chứng minh được gcd(x3+1,x2+1)=1

$\Rightarrow$ x3+1 là lập phương của 1 số nguyên.

Như vậy, x3 và x3+1 là 2 số nguyên liên tiếp và đều là lập phương của các số nguyên, và theo giả thiết x là số lẻ nên suy ra x= -1

Từ đó thay vào giả thiết tìm được y= 0

Vậy cặp số (x, y) thỏa mãn bài là (0, -1)




#706224 [TOPIC] ÔN THI BẤT ĐẲNG THỨC $\boxed{\text{THPT CHUYÊN}}$...

Đã gửi bởi xuanhoan23112002 on 17-04-2018 - 21:56 trong Tài liệu - Đề thi

Bài 28(IMO 1961): Cho tam giác ABC có độ dài 3 cạnh là a, b, c và có diện tích là S. Chứng minh rằng: $a^2+b^2+c^2\geq 4\sqrt{3}S$




#706223 [TOPIC] ÔN THI BẤT ĐẲNG THỨC $\boxed{\text{THPT CHUYÊN}}$...

Đã gửi bởi xuanhoan23112002 on 17-04-2018 - 21:53 trong Tài liệu - Đề thi

Bài 27: Cho a, b, c là các số không âm thỏa mãn: $a+b+c=3$

Chứng minh rằng $\sqrt{a}+\sqrt{b}+\sqrt{c}\geq ab+bc+ca$




#706221 Dạng toán: Trò chơi

Đã gửi bởi xuanhoan23112002 on 17-04-2018 - 21:40 trong IQ và Toán thông minh

max khó




#706211 [TOPIC] ÔN THI BẤT ĐẲNG THỨC $\boxed{\text{THPT CHUYÊN}}$...

Đã gửi bởi xuanhoan23112002 on 17-04-2018 - 20:46 trong Tài liệu - Đề thi

Bài 26: Cho x, y, z là các số thực không âm thỏa mãn: $x+y+z=100$

Xác định giá trị lớn nhất của M =$11xy+3xz+2012yz$




#706210 Chứng minh MB vuông góc MN khó

Đã gửi bởi xuanhoan23112002 on 17-04-2018 - 20:40 trong Hình học

Bài này có thể sử dụng tích vô hướng của lớp 10




#706208 [TOPIC] SỐ HỌC ÔN TẬP THPT CHUYÊN TOÁN 10 NĂM HỌC 2018-2019

Đã gửi bởi xuanhoan23112002 on 17-04-2018 - 20:33 trong Tài liệu - Đề thi

Bài 35: Giải phương trình nghiệm nguyên: $y^3=x^5+x^3+x^2+1$ với x là số lẻ




#706205 [TOPIC] ÔN THI BẤT ĐẲNG THỨC $\boxed{\text{THPT CHUYÊN}}$...

Đã gửi bởi xuanhoan23112002 on 17-04-2018 - 20:20 trong Tài liệu - Đề thi

Bài 25: Cho $0\leq a, b, c \leq 2$ và a+b+c=3

1. Tìm giá trị nhỏ nhất và giá trị lớn nhất của M =$a^2+b^2+c^2$

2. Tìm giá trị nhỏ nhất và giá trị lớn nhất của N =$a^3+b^3+c^3$

3. Tìm giá trị nhỏ nhất của H =$\sqrt{ab}+\sqrt{bc}+\sqrt{ca}$

P/s: Mỗi câu là 1 bài toán riêng mình ghép chung thành 1 bài




#706088 [TOPIC] ÔN THI BẤT ĐẲNG THỨC $\boxed{\text{THPT CHUYÊN}}$...

Đã gửi bởi xuanhoan23112002 on 16-04-2018 - 22:14 trong Tài liệu - Đề thi

Bài 13: Cho a, b, c >0 và a+b+c=1. CMR: $5(a^2+b^2+c^2)\leq 6(a^3+b^3+c^3)+1$

Đẳng thức xảy ra khi nào?




#706080 [TOPIC] ÔN THI BẤT ĐẲNG THỨC $\boxed{\text{THPT CHUYÊN}}$...

Đã gửi bởi xuanhoan23112002 on 16-04-2018 - 21:55 trong Tài liệu - Đề thi

Bài 10: Cho $a,b,c$ là các số thực dương thỏa mãn $abc=1$. Chứng minh rằng $\frac{\sqrt{a^{4}+b^{4}}}{1+ab}+\frac{\sqrt{b^{4}+c^{4}}}{1+bc}+\frac{\sqrt{c^{4}+a^{4}}}{1+ac}\geq 3$

Bất đẳng thức của bạn sai rồi vế phải là $\frac{3}{\sqrt{2}}$




#706077 [TOPIC] ÔN THI BẤT ĐẲNG THỨC $\boxed{\text{THPT CHUYÊN}}$...

Đã gửi bởi xuanhoan23112002 on 16-04-2018 - 21:51 trong Tài liệu - Đề thi

$P=\frac{a}{{b + c}} + \frac{b}{{a + c}} + \frac{c}{{a + b}} + \frac{{3{\rm{a}}bc}}{{\left( {a + b} \right)\left( {b + c} \right)\left( {c + a} \right)}}$

$P-2=\frac{(a+b-c)(a+c-b)(a-b-c)}{{abc(a+b)(b+c)(c+a)}}\geq 0$ Luôn đúng

Vậy $minP=2$ khi $a=b=c$

Quote : Không biết lời giải của mình có trùng với lời giải gốc không

Bạn ơi điều kiện ở bài này là $a\geq b+c$




#706068 Topic ôn thi hình học vào cấp 3 chuyên

Đã gửi bởi xuanhoan23112002 on 16-04-2018 - 21:14 trong Hình học

Bài 19 (Đề thi vào lớp 10 chuyên Nam Định 2017):Cho tam giác ABC có ba góc nhọn nội tiếp đường tròn (O), AB < AC. Các tiếp tuyến của đường tròn (O) tại B và C cắt nhau tại M. Đường thẳng qua M song song với AB cắt đường tròn (O) tại D và E (D thuộc cung nhỏ BC), cắt BC tại F, cắt AC tại I.

1) Chứng minh năm điểm M, B, O, I, C cùng thuộc một đường tròn

2)Chứng minh$\frac{FI}{FE}=\frac{FD}{FM}$

3) OI cắt (O) tại P và Q (P thuộc cung nhỏ AB). QF cắt (O) tại T (T khác Q). Tính tỉ số $\frac{TQ^2+TM^2}{MQ^2}$




#706061 [TOPIC] SỐ HỌC ÔN TẬP THPT CHUYÊN TOÁN 10 NĂM HỌC 2018-2019

Đã gửi bởi xuanhoan23112002 on 16-04-2018 - 20:36 trong Tài liệu - Đề thi

Bài 18:Giải phương trình nghiệm nguyên:$\frac{x^7-1}{x-1}=y^5-1$




#706058 [TOPIC] ÔN THI BẤT ĐẲNG THỨC $\boxed{\text{THPT CHUYÊN}}$...

Đã gửi bởi xuanhoan23112002 on 16-04-2018 - 20:21 trong Tài liệu - Đề thi

Bài 9: Đặt a+b+c=p,ab+bc+ca=q,abc=r

Theo giả thiết, $ =>p+r=4=>r=4=> p+r==> r=4p $ , từ giả thiết ta dễ dàng chứng minh được p$\geq$3

Theo bất đẳng thức Schur, ta có

$p^3-4pq + 9r \geq 0 => p^3-4pq + 9(4-p) \geq 0 =>  p^3- 9p+36 \geq 4pq => \frac{p^3-9p+36}{4p} \geq q$

$\Rightarrow \frac{p^3 -9p+36}{4p}\geq q$

Ta sẽ chứng minh p$\geq q$ hay ta chứng minh:p \geq \frac{p^3-9p+36}{4p}  <=> p^3 - 4p^2 -9p + 36 \leq 0 <=> 3 \leq p \leq 4(bất đẳng thức này hiển nhiên đúng)

Từ đó ta suy ra q$\leq$ 4

                     Vậy MaxP=4. Đẳng thức xảy ra chẳng hạn khi (a,b,c)=(0,2,2)
Bài 10: Cho a, b, c là các số thực không âm thỏa mãn $a^2+b^2+c^2+abc$=4
Tìm min và max của P=a+b+c



#705630 Số học

Đã gửi bởi xuanhoan23112002 on 12-04-2018 - 20:09 trong Số học

Tìm các số nguyên dương n sao cho: Với mọi a, b là các số nguyên dương, nếu a2b+1 chia hết cho n thì a2+b cũng chia hết cho n




#705432 $\frac{1}{a^2+2b^2+3}+\frac{1}...

Đã gửi bởi xuanhoan23112002 on 10-04-2018 - 21:59 trong Bất đẳng thức và cực trị

Từ giả thiết suy ra abc<=1

a2+2b2+3>=2ab+2b+2>0

$\frac{1}{a2+2b2+3}$<=$\frac{1}{2ab+2b+2}$

Làm tương tự như trên với các phân thức còn lại cùng với abc<=1 ta có điều phải chứng minh




#705345 Cần lắm một lời giải thích !

Đã gửi bởi xuanhoan23112002 on 09-04-2018 - 20:48 trong Hình học phẳng

Ý mk hỏi [mod $\pi$] nghĩa là gì

góc định hướng giữa 2 đường thẳng hơn nhau 1 bội của π

 




#705303 Cần lắm một lời giải thích !

Đã gửi bởi xuanhoan23112002 on 08-04-2018 - 23:22 trong Hình học phẳng

Đó là tính chất góc định hướng được tạo bởi 4 điểm đồng viên




#705302 ĐỀ THI OLYMPIC 30/4 NĂM 2018 THPT LHP TP.HCM - KHỐI 10

Đã gửi bởi xuanhoan23112002 on 08-04-2018 - 23:13 trong Thi HSG cấp Tỉnh, Thành phố. Olympic 30-4. Đề thi và kiểm tra đội tuyển các cấp.

Bài 4:

Theo đề bài ta có: $p^2 -p+1=x^3$(x là số tự nhiên,x>1)

Hay $p(p-1)=(x-1)(x^2+x+1)$

Do $p$ là số nguyên tố nên $x-1$ hoặc $x^2+x+1$ chia hết cho$ p$

Nếu $x-1$ chia hết cho $p$ thì $x-1\ge p,x^2+x+1<p$ (vô lí với x là số tự nhiên >1) 

Do đó $x^2+x+1$ chia hết cho $p$ nên $x^2+x+1=pk$ (k là số tự nhiên)

Ta xét$ k=1,2$ không thỏa mãn

Xét $k\ge 3$

Thay vào phương trình ta được:$ p-1=(x-1)k$ hay $p=(x-1)k+1$

Từ đó ta có: $x^2+x+1=(xk-k+1)k$

Hay $x^2+x(1-k^2)+k^2-k+1=0$

Đây là phương trình bậc 2 ẩn x, để phương trình có nghiệm tự nhiên thì $\delta =k^4-6k^2+4k-3$ phải là số chính phương

Ta có: $(k^2-3)^2\le k^4-6k^2+4k-3<(k^2-2)^2$

Từ đó ta tìm được k=3 ta tìm được x=7, p=19 là số nguyên tố

Vậy p=19 là số nguyên tố thỏa mãn đề bài




#705205 Tìm a? đề hàm số có đạo hàm tại x=-1

Đã gửi bởi xuanhoan23112002 on 08-04-2018 - 08:32 trong Hàm số - Đạo hàm

Ta phải xét tính liên tục của hàm số và đặt điều kiện đạo hàm trái = đạo hàm phải bạn nhé




#705203 Trong một giải đấu bóng đá có 10 đội tham gia theo thể thức mỗi đội đều gặp đ...

Đã gửi bởi xuanhoan23112002 on 08-04-2018 - 08:27 trong Tổ hợp và rời rạc

Ta chứng minh bài toán bằng phản chứng( cả 1 và 2 đều không xảy ra)

Gọi 10 đội bóng là a(i là số tự nhiên và i chạy từ 1 đến 10)

Giả sử a10 là đội bóng có số trận thua nhiều nhất

Khi đó nếu tồn tại giá trị i từ 1 đến 9 mà a10 thang ai thì tất cả cả đội bóng mà a10 thua thì ai cũng thua (vô lí do a10 có số trận thua nhiều nhất)

Suy ra a10 thi đấu với các đội còn lại chỉ có thể hòa hoặc thua

Mà theo gia sư điều kiện 2 không xảy ra nên a10 thua ít nhất 7 đội là aj (j chạy từ 1 đến 7)

Lập luận tương tự như trên với a7 là đội có số trận thua nhiều nhất trong 7 đội trên thì a7 phải thừa ít nhất 4 đội giả sử là: a1, a2, a3, a4.

Lập luận tương tự như trên với a4 là đội có số trận thua nhiều nhất trong 4 đội trên thì a4 phải thừa ít nhất 1 đội giả sử là: a1

Như vậy ta tìm được 4 đội: a1, a4, a7, a10, lập thành 4 đội thỏa mãn điều kiện 1( mâu thuẫn với giả sử)

Do đó giả sử sai. Ta có điều phải chứng minh




#705140 chứng minh

Đã gửi bởi xuanhoan23112002 on 06-04-2018 - 23:36 trong Số học

Lời giải nhanh nhất cho bài toán này là cách áp dụng định lý Fermat nhỏ:

x- x chia hết cho 5

Dễ dàng chứng minh x5 - x chia hết cho 2

Như vậy ta cũng có điều phải chứng minh




#705137 chứng minh

Đã gửi bởi xuanhoan23112002 on 06-04-2018 - 23:24 trong Số học

chứng minh rằng với mọi n$\epsilon$$\mathbb{Z}$ thì $n^{5}$ và n có chữ số tận cùng giống nhau 

n5 - n =n(n-1)(n+1)(n2+1)

Dễ dàng chứng minh n5-n chia hết cho 2

Với n chia hết cho 5, chia 5 dư 1 hoặc 4 thì n5-n chia hết cho 5

Với n chia 5 dư 2 hoặc 3 thì n2+1 chia hết cho 5

Như vậy ta có n5-n chia hết cho 5, n5-n chia hết cho 2 và gcd(5,2)=1

Nên n5-n chia hết cho 10

Hay n5 và n có chữ số tận cùng giống nhau




#705136 Chứng minh rằng với mọi số thực dương $a,b>2$ thì $2^a-1...

Đã gửi bởi xuanhoan23112002 on 06-04-2018 - 22:59 trong Số học

Bài toán sai khi a chia hết cho b 

Nếu a không chia hết cho b. Đặt a=bq+r(0<r<b)

Sử dụng phản chứng để suy ra 2r - 1 chia hết cho 2b - 1 (điều này vô lí do 0<2r - 1<2b - 1)

Từ đó ta có điều phải chứng minh.